0

De moivre’s theorem and its applications - class-XII

Description: de moivre’s theorem and its applications
Number of Questions: 38
Created by:
Tags: maths demoivre's theorem complex numbers
Attempted 0/38 Correct 0 Score 0

If $iz^4 + 1 = 0$, then z can take the value

  1. $\displaystyle \frac{1 + i}{\sqrt 2}$

  2. $\cos \displaystyle \frac{\pi}{8} + i \sin \frac{\pi}{8}$

  3. $\displaystyle \frac{1}{4 i}$

  4. $i$


Correct Option: B
Explanation:
$iz^4 + 1 = 0 \Rightarrow z^4 = - \displaystyle \frac{1}{i} = \frac{i^2}{i} = i$
Let $z^4 = \cos  \displaystyle \frac{\pi}{2} + i  \sin  \frac{\pi}{2}$
$\therefore z = \displaystyle \left [ \cos  \frac{\pi}{2} + i  \sin  \frac{\pi}{2} \right ]^{1/4}$
U\sin g De-Moivre's theorem,
$(\cos   \theta + i  \sin  \theta)^n = \cos   n \theta + i  \sin   n \theta, n  \varepsilon I$
Hence, $z = \cos  \displaystyle \frac{\pi}{8} + i  \sin  \frac{\pi}{8}$

The product of the values of $\displaystyle{\left[ {\cos {\pi  \over 3} + i\sin {\pi  \over 3}} \right]^{{3 \over 4}}}$ is

  1. $-1$

  2. $1$

  3. $i$

  4. $-i$


Correct Option: B
Explanation:

Given: $\displaystyle{\left[ {\cos {\pi  \over 3} + i\sin {\pi  \over 3}} \right]^{{3 \over 4}}}$


$=[e^{i(\pi/3)}]^{(3/4)}=e^{i\pi(1/3)(3/4)}=e^{4\pi i}=cos4\pi+isin{4\pi}=1-0i=1$ 

Number of integral values of n for which the quantity ${n+i}^{4}$ where ${i}^{2}=-1$, is an integer is

  1. $1$

  2. $2$

  3. $3$

  4. Infinite


Correct Option: D
Explanation:

Simply $n+i^{4}$ = $n+1$ which is integer for every integral value of n .

So there are infinite values of n .

De Moivre's theorem

$(\cos\theta +i\sin \theta )=\cos n\theta $ if n is an integer and $\cos n\theta +i \sin n\theta $ is one of the values of $(\cos\theta +i\sin\theta )^{n}$, if n is a fraction.

Corollary : The q values of ($(\cos\theta +i\sin\theta )^{\frac{1}{q}}$ are obtained from

cos $\frac{2n\pi +\theta }{q}+i\sin\frac{2n\pi +\theta }{q}$ by putting n = 0, 1, 2, ..., (q - 1).


  1. Both are correct

  2. Only first statement is true.

  3. Only second ststement is true

  4. None 


Correct Option: A

If $a = {\mathop{\rm cis}\nolimits} \alpha ,b = cis\beta ,c = cis\gamma $ then $\dfrac{{{a^3}{b^3}}}{{{c^2}}} = $

  1. $cis(3\alpha + 3\beta + 2\gamma )$

  2. $cis(3\alpha + 3\beta - 2\gamma )$

  3. $cis( - 3\alpha - 3\beta + 2\gamma )$

  4. $cis(3\alpha - 3\beta + 2\gamma )$


Correct Option: B
Explanation:

$a=cis\alpha=Cos\alpha+iSin\alpha= \ e^{i\alpha}$

$b=cis\beta=Cos\beta+iSin\beta= \ e^{i\beta}$      

$a=cis\gamma=Cos\gamma+iSin\gamma= \ e^{i\gamma}$

$\therefore \dfrac{a^3b^3}{c^2}=\dfrac{({e^{i\alpha}})^3({e^{i\beta}})^3}{({e^{i\gamma}})^2}$

$=\dfrac{e^{3i\alpha}e^{3i\beta}} {e^{2i\gamma}}=\ e^{i(3\alpha+3\beta-2\gamma)}$

$=Cos(3\alpha+3\beta-2\gamma)+iSin(3\alpha+3\beta-2\gamma)$

$=cis(3\alpha+3\beta-2\gamma)$

If $a=\cos { \left( \cfrac { 8\pi  }{ 11 }  \right)  } +i\sin { \left( \cfrac { 8\pi  }{ 11 }  \right)  } $, then $Re(a+{a}^{2}+{a}^{3}+{a}^{4}+{a}^{5})=$

  1. $0$

  2. $-\cfrac{1}{2}$

  3. $\cfrac{1}{2}$

  4. $1$


Correct Option: B

For ${ Z } _{ 1 }=\sqrt [ 6 ]{ \dfrac { 1-i }{ 1+i\sqrt { 3 }  }  } $, ${ Z } _{ 2 }=\sqrt [ 6 ]{ \dfrac { 1-i }{ \sqrt { 3 } +i }  } $, ${ Z } _{ 3 }=\sqrt [ 6 ]{ \dfrac { 1+i }{ \sqrt { 3 } -i }  } $ which of the following holds goods?

  1. $\sum { { \left| { Z } _{ 1 } \right| }^{ 2 } } =\dfrac { 3 }{ 2 } $

  2. ${ \left| { Z } _{ 1 } \right| }^{ 4 }+{ \left| { Z } _{ 2 } \right| }^{ 4 }={ \left| { Z } _{ 3 } \right| }^{ -8 }$

  3. $\sum { { \left| { Z } _{ 1 } \right| }^{ 3 }+{ \left| { Z } _{ 2 } \right| }^{ 3 }={ \left| { Z } _{ 3 } \right| }^{ -6 } } $

  4. $\ \ \ { \left| { Z } _{ 1 } \right| }^{ 4 }+{ \left| { Z } _{ 2 } \right| }^{ 4 }={ \left| { Z } _{ 3 } \right| }^{ 8 }$


Correct Option: A

Given z is a complex number with modulus 1. Then the equation $\left[\dfrac{(1+ia)}{(1-ia)}\right]^4$ = z has

  1. all roots real and distinct

  2. two real and one imaginary

  3. three roots real and one imaginary

  4. one root real and three imaginary


Correct Option: A
Explanation:

$\displaystyle { \left( \frac { 1+ia }{ 1-ia }  \right)  }^{ 4 }=z\quad $         ...(1)
$\displaystyle & \quad \left| z \right| =1$
$\displaystyle z=cisA=\cos { A } +i\sin { A } $
substitute z in equation (1)
$\displaystyle { \left( \frac { 1+ia }{ 1-ia }  \right)  }={ cisA }^{ \frac { 1 }{ 4 }  }=cis\frac { 2k\pi +A }{ 4 } $       ...{De Moivre's Theorem}
where $k=0,1,2,3$

Let $\displaystyle B=\frac { 2k\pi +A }{ 4 } $

$\displaystyle \Longrightarrow ia=\frac { -1+cisB }{ 1+cisB } =\frac { \sin { \frac { B }{ 2 } \left( i\cos { \frac { B }{ 2 }  } -\sin { \frac { B }{ 2 }  }  \right)  }  }{ \cos { \frac { B }{ 2 } \left( \cos { \frac { B }{ 2 }  } +i\sin { \frac { B }{ 2 }  }  \right)  }  } $

$\displaystyle \Longrightarrow ia=\frac { i\sin { \frac { B }{ 2 } \left( \cos { \frac { B }{ 2 }  } +i\sin { \frac { B }{ 2 }  }  \right)  }  }{ \cos { \frac { B }{ 2 } \left( \cos { \frac { B }{ 2 }  } +i\sin { \frac { B }{ 2 }  }  \right)  }  } $

$\displaystyle \Longrightarrow a=\tan { \frac { B }{ 2 }  } $

Therefore roots are real and distinct.

Ans: A

If $\sqrt{5 - 12i} + \sqrt{-5 - 12i} = z$, then principal value of arg z can be 

  1. $-\displaystyle\frac{\pi}{4}$

  2. $\displaystyle\frac{\pi}{4}$

  3. $-\displaystyle\frac{3\pi}{4}$

  4. $\displaystyle\frac{3\pi}{4}$


Correct Option: A,B,C,D
Explanation:

Dividing and multiplying by $\sqrt{13}$
$=\sqrt{13}[(\dfrac{5}{13}-\dfrac{12i}{13})^{\dfrac{1}{2}}+(-\dfrac{5}{13}-\dfrac{12i}{13})^{\dfrac{1}{2}}]$
$=\sqrt{13}[e^{i\dfrac{-\theta}{2}}+e^{i\dfrac{\theta-\pi}{2}}]$
$=\sqrt{13}[cos\dfrac{\theta}{2}-isin\dfrac{\theta}{2}+sin\dfrac{\theta}{2}-icos\frac{\theta}{2}]$
$=\sqrt{13}[cos\dfrac{\theta}{2}+sin\dfrac{\theta}{2}-i(sin\dfrac{\theta}{2}+cos\dfrac{\theta}{2})]$
$=z$
Here $\theta=sin^{-1}(\dfrac{12}{13})$
Hence
$|Re(z)|=|Im(z)|$
Hence argument of $Z$ is in the form of $\dfrac{2n-1(\pi)}{4}$ $n\epsilon::Integers$

The value of $\displaystyle { \left( \frac { 1+i }{ \sqrt { 2 }  }  \right)  }^{ 8 }+{ \left( \frac { 1-i }{ \sqrt { 2 }  }  \right)  }^{ 8 }$ is equal to

  1. $4$

  2. $6$

  3. $8$

  4. $2$


Correct Option: D
Explanation:

We have $\displaystyle { \left( \frac { 1+i }{ \sqrt { 2 }  }  \right)  }^{ 8 }+{ \left( \frac { 1-i }{ \sqrt { 2 }  }  \right)  }^{ 8 }$


$\displaystyle={ \left[ \cos { \frac { \pi  }{ 4 }  } +i\sin { \frac { \pi  }{ 4 }  }  \right]  }^{ 8 }+{ \left[ \cos { \frac { \pi  }{ 4 }  } -i\sin { \frac { \pi  }{ 4 }  }  \right]  }^{ 8 }$


$=\cos { 2\pi  } +i\sin { 2\pi  } +\cos { 2\pi  } -i\sin { 2\pi  } $      [by de-moivre's theorem]

$=2\cos { 2\pi  } =2\left( 1 \right) =2$  

The number of solutions of equation $z^{10}-z^{5}+1=0$ are 

  1. only two solution

  2. No solution

  3. only five solution

  4. exactly 10


Correct Option: D
Explanation:

${ z }^{ 10 }-{ z }^{ 5 }+1=0$

Let ${ z }^{ 5 }=w$
$\Rightarrow { w }^{ 2 }-w+1=0$

$\Rightarrow w=\frac { 1\pm \sqrt { 3 }  }{ 2 } =\cos { \frac { \Pi  }{ 3 }  } \pm \sin { \frac { \Pi  }{ 3 }  } =cis\left( \pm \frac { \Pi  }{ 3 }  \right) $


$\Rightarrow { z }^{ 5 }=cis\left( \pm \frac { \Pi  }{ 3 }  \right) $

Case 1:
${ z }^{ 5 }=cis\left( \frac { \Pi  }{ 3 }  \right) $

$\Rightarrow z={ \left( cis\left( \frac { \Pi  }{ 3 }  \right)  \right)  }^{ \frac { 1 }{ 5 }  }=cis\left( \frac { 2k\Pi +\Pi  }{ 15 }  \right) \ $        ...{De Moivre's Theorem}

Where k=0,1,2,3,4.
Therefore number of solutions are 5.

Case 2:
${ z }^{ 5 }=cis\left( -\frac { \Pi  }{ 3 }  \right) $

$\Rightarrow z={ \left( cis\left( -\frac { \Pi  }{ 3 }  \right)  \right)  }^{ \frac { 1 }{ 5 }  }=cis\left( \frac { 2k\Pi -\Pi  }{ 15 }  \right) $       ...{De Moivre's Theorem}

Where k=0,1,2,3,4.
Therefore number of solutions are 5.

From case 1 & case 2 total number of solutions of equation ${ z }^{ 10 }-{ z }^{ 5 }+1=0$ are 10.

Ans: D

If $\displaystyle z=1+\cos \frac{2\pi }{3}+i\sin \frac{2\pi }{3}$, then

  1. $\displaystyle Re(z^{5})=\frac{\sqrt{3}}{2}$

  2. $\displaystyle Re(z^{5})=\frac{1}{2}$

  3. $\displaystyle Im(z^{5})=\frac{1}{2}$

  4. $\displaystyle Im(z^{5})=\frac{\sqrt{3}}{2}$


Correct Option: B
Explanation:

$z=1+\cos { \frac { 2\pi  }{ 3 }  } +i\sin { \frac { 2\pi  }{ 3 }  } =2\cos ^{ 2 }{ \frac { \pi  }{ 3 }  } +2i\sin { \frac { \pi  }{ 3 } \cos { \frac { \pi  }{ 3 }  }  } $

$\displaystyle \Rightarrow z=2\cos { \frac { \pi  }{ 3 }  } \left( \cos { \frac { \pi  }{ 3 }  } +i\sin { \frac { \pi  }{ 3 }  }  \right) =\cos { \frac { \pi  }{ 3 }  } +i\sin { \frac { \pi  }{ 3 }  } $

$\displaystyle \Rightarrow { z }^{ 5 }={ \left( \cos { \frac { \pi  }{ 3 }  } +i\sin { \frac { \pi  }{ 3 }  }  \right)  }^{ 5 }=\cos { \frac { 5\pi  }{ 3 }  } +i\sin { \frac { 5\pi  }{ 3 }  } $        ...{De Moivre's Theorem}
 
$\displaystyle \Rightarrow { z }^{ 5 }=\frac { 1-i\sqrt { 3 }  }{ 2 } $

$\displaystyle \therefore \quad Re\left( { z }^{ 5 } \right) =\frac { 1 }{ 2 } \quad & \quad Im\left( { z }^{ 5 } \right) =\frac { -\sqrt { 3 }  }{ 2 } $
Hence, option B is correct.

Construct an equation whose roots are $n^{th}$ powers of the roots of the equation $\displaystyle x^{2}-2x\cos \theta +1= 0.$

  1. $\displaystyle x^{2}-2n\cos n\theta x+1= 0$

  2. $\displaystyle x^{2}-2n\cos \theta x+1= 0$

  3. $\displaystyle x^{2}-2\cos n\theta x+1= 0$

  4. $\displaystyle x^{2}-2\cos ^{n}\theta x+1= 0$


Correct Option: C
Explanation:

We know, $\displaystyle \alpha = \cos \theta +i\sin \theta , \beta = \cos \theta -i\sin \theta $
$\displaystyle \alpha ^{n}= \cos n\theta +i\sin n\theta ,$
$\displaystyle \beta ^{n}= \cos n\theta -i\sin n\theta $
$\displaystyle S= 2\cos n\theta , P= 1 \therefore x^{2}-Sx+P= 0$
or $\displaystyle x^{2}-2\cos n\theta x+1= 0$ is the required equation.

Ans: C

If $z = \left(\displaystyle\frac{\sqrt3}{2} + \displaystyle\frac{i}{2}\right)^{2009}+\left(\displaystyle\frac{\sqrt3}{2} - \displaystyle\frac{i}{2}\right)^{2009}$, then 

  1. $Im(z) = 0$

  2. $Re(z) > 0$

  3. $Im(z) > 0$

  4. $Re(z) < 0, Im(z) > 0$


Correct Option: A
Explanation:

As we know that,
$\dfrac { \sqrt { 3 }  }{ 2 } +\dfrac { i }{ 2 } =\cos { \dfrac { \pi  }{ 6 }  } +i\sin { \dfrac { \pi  }{ 6 }  } $

and $\dfrac { \sqrt { 3 }  }{ 2 } -\dfrac { i }{ 2 } =\cos { \dfrac { \pi  }{ 6 }  } -i\sin { \dfrac { \pi  }{ 6 }  } $

$z=\left( \dfrac { \sqrt { 3 }  }{ 2 } +\dfrac { i }{ 2 }  \right) ^{ 2009 }+\left( \dfrac { \sqrt { 3 }  }{ 2 } -\dfrac { i }{ 2 }  \right) ^{ 2009 }$

$\Rightarrow z=\left( \cos { \dfrac { \pi  }{ 6 }  } +i\sin { \dfrac { \pi  }{ 6 }  }  \right) ^{ 2009 }+\left( \cos { \dfrac { \pi  }{ 6 }  } -i\sin { \dfrac { \pi  }{ 6 }  }  \right) ^{ 2009 }$         ......{ De Moivre's Theorem}

$\Rightarrow z=\cos { \dfrac { 2009\pi  }{ 6 }  } +i\sin { \dfrac { 2009\pi  }{ 6 }  } +\cos { \dfrac { 2009\pi  }{ 6 }  } -i\sin { \dfrac { 2009\pi  }{ 6 }  } $

$\Rightarrow z=2\cos { \dfrac { 2009\pi  }{ 6 }  } $


Therefore, $Im(z)=0$

Ans: B

The roots of $\displaystyle \left ( -64a^{4} \right )^{\tfrac14}$ are

  1. $\displaystyle \pm 2a\left ( 1\pm i \right ).$

  2. $\displaystyle \pm a\left ( 1\pm i \right ).$

  3. $\displaystyle \pm 2a\left ( 1\pm 2i \right ).$

  4. $\displaystyle \pm a\left ( 1\pm 2i \right ).$


Correct Option: A
Explanation:
$\displaystyle \left ( -64a^{4} \right )^{\tfrac14}= \left ( 2\sqrt{2} \right )a\left ( -1 \right )^{\tfrac14}$
We know that $\displaystyle -1= \cos \pi +i\sin \pi $
Now put $\displaystyle -1= r\cos \theta , 0= r\sin \theta $
$\displaystyle \therefore \left ( -64a^{4} \right )^{\tfrac14}= 2\sqrt{2}a.\left [ \cos \pi +i\sin \pi  \right ]^{\tfrac14}$
$\displaystyle = 2\sqrt{2a}\left [ \cos \left ( 2n\pi +\pi  \right )+i\sin \left ( 2n\pi +\pi  \right ) \right ]^{\tfrac14}$
$\displaystyle = 2\sqrt{2a}\left [ \cos \cfrac{2n\pi +\pi }{4}+i\sin \cfrac{2n\pi +\pi }{4} \right ],$
where n=0, 1, 2 and 3.Hence the required roots are
$\displaystyle 2\sqrt{2}a\left [ \cos \left ( \cfrac{\pi}{4} \right )+i\sin \left ( \cfrac{\pi}{4} \right ) \right ],$
$\displaystyle 2\sqrt{2}a\left [ \cos \left ( 3\cfrac{\pi}{4} \right )+i\sin \left ( 3\cfrac{\pi}{4} \right ) \right ],$
$\displaystyle 2\sqrt{2}a\left [ \cos \left ( 5\cfrac{\pi}{4} \right )+i\sin \left ( 5\cfrac{\pi}{4} \right ) \right ],$
$\displaystyle 2\sqrt{2}a\left [ \cos \left ( 7\cfrac{\pi}{4} \right )+i\sin \left ( 7\cfrac{\pi}{4} \right ) \right ],$
Thus the roots on putting the values are
$\displaystyle 2\sqrt{2}a\left ( \dfrac{1}{\sqrt{2}}+\dfrac{i}{\sqrt{2}} \right ), 2\sqrt{2}a\left (\dfrac{-1}{\sqrt{2}}+\dfrac{i}{\sqrt{2}} \right ),$
$\displaystyle 2\sqrt{2}a\left ( \dfrac{-1}{\sqrt{2}}-\dfrac{i}{\sqrt{2}} \right ), 2\sqrt{2}a\left (\dfrac{1}{\sqrt{2}}-\dfrac{i}{\sqrt{2}} \right ).$
Hence the roots are $\displaystyle \pm 2a\left ( 1\pm i \right ).$

Ans: $A$

The value of $(iz+z^5+z^8)$ when $z=\dfrac{\sqrt{3}+i}{2}$ is?

  1. $0$

  2. $-1$

  3. $\dfrac{-\sqrt{3}+i}{2}$

  4. $z$


Correct Option: C
Explanation:

$z=\dfrac{\sqrt{3}+i}{2}=\cos \dfrac{\pi}{6}+i\sin\dfrac{\pi}{6}=cis\dfrac{\pi}{6}$


$iz=icis\dfrac{\pi}{6}=cis\dfrac{\pi}{3}$

$z^5=cis\dfrac{5\pi}{6}$

$z^8=cis\dfrac{8\pi}{6}=-cis\dfrac{\pi}{3}$

$\implies \left(cis\dfrac{5\pi}{6}\right)=\dfrac{-\sqrt{3}+i}{2}$ 

The value of $\displaystyle \left ( \sin \frac{\pi }{8}+i\cos \frac{\pi }{8} \right )^{8}$

  1. -1

  2. 1

  3. 0

  4. None of these


Correct Option: A
Explanation:

$z={ \left( \sin { \frac { \pi  }{ 8 } +i } \cos { \frac { \pi  }{ 8 }  }  \right)  }^{ 8 }={ \left[ i\left( \cos { \frac { \pi  }{ 8 } -i\sin { \frac { \pi  }{ 8 }  }  }  \right)  \right]  }]^8$
     ...{$\because \quad { i }^{ 8 }=1$}
$\Rightarrow z=\cos { \pi -i\sin { \pi  }  } =-1$        ...{De Moivre's Theorem}
Hence, option 'A' is correct.

If $z=\cos 2\theta +i\sin 2\theta $ then which is correct 

  1. $\displaystyle \sum _{r=0}^{n}C _{r}\cos2r\theta =2^{n} \cos ^{n}\theta \cos n\theta $

  2. $\displaystyle \sum _{r=1}^{n}C _{r}\cos2r\theta =2^{n} \sin ^{n}\theta \cos n\theta $

  3. $\sum _{ r=0 }^{ n } C _{ r }\sin  2r\theta =2^{ n }\cos ^{ n } \theta \sin  n\theta $

  4. $\displaystyle \sum _{r=0}^{n}C _{r}\sin2r\theta =2^{n} \sin ^{n}\theta \sin n\theta $


Correct Option: A,C
Explanation:

By Binomial Theorem
${ \left( 1+z \right)  }^{ n }={ C } _{ 0 }+{ C } _{ 1 }z+{ C } _{ 2 }{ z }^{ 2 }+{ C } _{ 3 }{ z }^{ \ 3 }+....+{ C } _{ n }{ z }^{ n }=\sum _{ r=0 }^{ n }{ { C } _{ r }{ z }^{ r } } $      ...(1)

Substituting $z=\cos { 2\theta  } +i\sin { 2\theta  }  $ in eq. (1), we get

${ \left( 1+\cos { 2\theta  } +i\sin { 2\theta  }  \right)  }^{ n }=\sum _{ r=0 }^{ n }{ { C } _{ r }\left( \cos { 2\theta  } +i\sin { 2\theta  }  \right) ^{ r } } $

$\Rightarrow \sum _{ r=0 }^{ n }{ { C } _{ r }\left( \cos { 2r\theta  } +i\sin { 2r\theta  }  \right)  }$      ...{De Moivre's Theorem}

$\Rightarrow { \left[ 2\cos { \theta  } \left( \cos { \theta  } +i\sin { \theta  }  \right)  \right]  }^{ n }=\sum _{ r=0 }^{ n }{ { C } _{ r }\cos { 2r\theta  }  } +i\sum _{ r=0 }^{ n }{ { C } _{ r }\sin { 2r\theta  }  } $

$\Rightarrow \sum _{ r=0 }^{ n }{ { C } _{ r }\cos { 2r\theta  }  } +i\sum _{ r=0 }^{ n }{ { C } _{ r }\sin { 2r\theta  }  } ={ 2 }^{ n }\cos ^{ n }{ \theta  } { \left( \cos { n\theta  } +i\sin { n\theta  }  \right)  }$         ...{De Moivre's Theorem}

$\Rightarrow \sum _{ r=0 }^{ n }{ { C } _{ r }\cos { 2r\theta  }  } +i\left( \sum _{ r=0 }^{ n }{ { C } _{ r }\sin { 2r\theta  }  }  \right) ={ 2 }^{ n }\cos ^{ n }{ \theta  } \cos { n\theta  } +i\left( { 2 }^{ n }\cos ^{ n }{ \theta  } \sin { n\theta  }  \right) $

On comparing real and Imaginary parts, we get
$\sum _{ r=0 }^{ n }{ { C } _{ r }\cos { 2r\theta  }  } ={ 2 }^{ n }\cos ^{ n }{ \theta  } \cos { n\theta  } \quad &amp; \quad \sum _{ r=0 }^{ n }{ { C } _{ r }\sin { 2r\theta  }  } ={ 2 }^{ n }\cos ^{ n }{ \theta  } \sin { n\theta  } $
Hence, option 'A' and 'C' are correct.

Put in the form  A +iB

$\displaystyle \frac{\left ( \cos 2\theta -i\sin 2\theta  \right )^{7}\left ( \cos 3\theta +i\sin 3\theta  \right )^{-5}}{\left ( \cos 4\theta +i\sin 4\theta  \right )^{12}\left ( \cos 5\theta +i\sin 5\theta  \right )^{-6}}$

  1. $\displaystyle\cos 47\theta +i\sin47\theta.$

  2. $\displaystyle\cos 47\theta -i\sin47\theta.$

  3. $\displaystyle\cos 41\theta +i\sin41\theta.$

  4. $\displaystyle\cos 41\theta -i\sin41\theta.$


Correct Option: B
Explanation:

Using De-Moivre's Theorem, the given expression
$\displaystyle = \frac{\left ( \cos \theta -i\sin \theta  \right )^{14}\left ( \cos \theta +i\sin \theta  \right )^{-15}}{\left ( \cos \theta +i\sin \theta  \right )^{48}\left ( \cos \theta +i\sin \theta  \right )^{-30}}$
$\displaystyle=\frac{\left ( e^{i\theta } \right )^{-29}}{\left ( e^{i\theta } \right )^{18}}=\left ( e^{i\theta } \right )^{-47}$
$\displaystyle=\left ( \cos \theta +i\sin \theta  \right )-^{47}=\cos 47\theta -\sin47\theta.$ 

Ans: B

If $z = \left(\displaystyle\frac{\sqrt3}{2}+\displaystyle\frac{i}{2}\right)^5 + \left(\displaystyle\frac{\sqrt3}{2}-\displaystyle\frac{i}{2}\right)^5,$ then

  1. $Re(z) = 0$

  2. $Im(z) = 0$

  3. $Re(z) > 0, \space Im(z) > 0$

  4. $Re(z) > 0, \space Im(z) < 0$


Correct Option: B
Explanation:

As we know that,
$\dfrac { \sqrt { 3 }  }{ 2 } +\dfrac { i }{ 2 } =\cos { \dfrac { \pi  }{ 6 }  } +i\sin { \dfrac { \pi  }{ 6 }  } $
and $\dfrac { \sqrt { 3 }  }{ 2 } -\dfrac { i }{ 2 } =\cos { \dfrac { \pi  }{ 6 }  } -i\sin { \dfrac { \pi  }{ 6 }  } $

$z=\left( \dfrac { \sqrt { 3 }  }{ 2 } +\dfrac { i }{ 2 }  \right) ^{ 5 }+\left( \dfrac { \sqrt { 3 }  }{ 2 } -\dfrac { i }{ 2 }  \right) ^{ 5 }$

$\Rightarrow z=\left( \cos { \dfrac { \pi  }{ 6 }  } +i\sin { \dfrac { \pi  }{ 6 }  }  \right) ^{ 5 }+\left( \cos { \dfrac { \pi  }{ 6 }  } -i\sin { \dfrac { \pi  }{ 6 }  }  \right) ^{ 5 }$         ......{ De Moivre's Theorem}

$\Rightarrow z=\cos { \dfrac { 5\pi  }{ 6 }  } +i\sin { \dfrac { 5\pi  }{ 6 }  } +\cos { \dfrac { 5\pi  }{ 6 }  } -i\sin { \dfrac { 5\pi  }{ 6 }  } $

$\Rightarrow z=-\dfrac { \sqrt { 3 }  }{ 2 } $
Therefore, $Im(z)=0$

Ans: B

If $z + z^{-1} = 1$, then $z^{100} + z^{-100}$ is equal to

  1. $i$

  2. $-i$

  3. $1$

  4. $-1$


Correct Option: D
Explanation:
If $z+z^{-1}=1,$ then $z^{100}+z^{-100}$
$\Rightarrow z+z^{-1}=1,$ when we multiply by $z$
$\Rightarrow z^2+1=z$
$\Rightarrow z^2-z+1=0$
By solving, $z=\dfrac { 1\pm \sqrt { 1-4 }  }{ 2 } =\dfrac { 1\pm i\sqrt { 3 }  }{ 5 } $
In polar form : $z=re^{iQ},$
$\Rightarrow r^2={ \left( \dfrac { 1 }{ 2 }  \right)  }^{ 2 }{ \left( \dfrac { \sqrt { 3 }  }{ 2 }  \right)  }^{ 2 }=\dfrac{1}{4}+\dfrac{3}{4}=1$
$\therefore r=1$
$\Rightarrow \tan \theta \dfrac { \pm \dfrac { \sqrt { 3 }  }{ 2 }  }{ \dfrac { 1 }{ 2 }  } =\pm \sqrt { 3 } $ i.e, $\theta =\pm \dfrac { \pi  }{ 3 } $ or $\pm \dfrac { 2\pi  }{ 3 } $
$\therefore z={ e }^{ \pm i{ \pi  }/{ 3 } }$ and $z^{-1}={ e }^{ \pm i{ \pi  }/{ 3 } }$
then $z^{100}={ e }^{ \pm i{ 100  }/{ 3 }\pi }=z^{\pm i\left(16\pi+\pi+1/3\pi\right)}$
$={ e }^{ \pm i{ \pi  }/{ 3 } }=-z$
$\therefore z^{-100}=-z^{-1}$
$\Rightarrow z^{100}+z^{-100}=-z-z^{-1}=-\left(z+z^{-1}\right)=-1$
Hence, the answer is $-1.$

The modulus and amplitude of the complex number $[e^{3-i \tfrac{\pi}{4}}]^3$ are respectively.

  1. $e^9, \dfrac{\pi}{2}$

  2. $e^9, \dfrac{-\pi}{2}$

  3. $e^6, \dfrac{-3\pi}{4}$

  4. $e^9, \dfrac{-3\pi}{4}$


Correct Option: D
Explanation:

$z=(e^{3-\tfrac{i\pi}{4}})^{3}$
$=(e^{3}.e^{-\tfrac{i\pi}{4}})^{3}$
$=e^{9}.e^{-\tfrac{3i\pi}{4}}$
$=|z|e^{i\arg(z)}$ 
By comparing RHS and LHS we get
$|z|=e^{9}$ and $\arg(z)=\dfrac{-3\pi}{4}$.

If $\displaystyle\alpha =\cos { \left( \frac { 8\pi  }{ 11 }  \right)  } +i\sin { \left( \frac { 8\pi  }{ 11 }  \right)  } ,$ then $Re\left( \alpha +{ \alpha  }^{ 2 }+{ \alpha  }^{ 3 }+{ \alpha  }^{ 4 }+{ \alpha  }^{ 5 } \right) $ is equal to

  1. $\displaystyle\frac { 1 }{ 2 } $

  2. $\displaystyle-\frac { 1 }{ 2 } $

  3. $0$

  4. None of these


Correct Option: B
Explanation:

$\displaystyle\alpha =\cos { \left( \frac { 8\pi  }{ 11 }  \right)  } +i\sin { \left( \frac { 8\pi  }{ 11 }  \right)  } $


We know, $z=\cos\theta+i\sin\theta=e^{i\theta}$
$\therefore$  $\alpha=e^{i\frac{8\pi}{11}}$
$\Rightarrow$  $\alpha+\alpha^2+\alpha^3+\alpha^4+\alpha^5=\dfrac{\alpha(\alpha^5-1)}{\alpha-1}$             ........................... as forming G.P.

                                                  $=\dfrac{\alpha^6-\alpha}{\alpha-1}$

                                                  $=\dfrac{\left(e^{i\frac{8\pi}{11}}\right)^6-e^{i\frac{8\pi}{11}}}{e^{i\frac{8\pi}{11}}-1}$           ---- ( 1 )

$e^{-\frac{48\pi}{11}}=\cos\dfrac{48\pi}{11}+i\sin\dfrac{48\pi}{11}$

          $=\cos\left(4\pi+\dfrac{4\pi}{11}\right)+i\sin\left(4\pi+\dfrac{4\pi}{11}\right)$

          $=\cos\dfrac{4\pi}{11}+i\sin\dfrac{4\pi}{11}$

          $=e^{i\frac{4\pi}{11}}$

Substituting above value in ( 1 ) we get,
$\Rightarrow$  $\alpha+\alpha^2+\alpha^3+\alpha^4+\alpha^5=\dfrac{e^{i\frac{4\pi}{11}}-e^{i\frac{8\pi}{11}}}{e^{i\frac{8\pi}{11}}-1}$

                                                  $=\dfrac{t-t^2}{t^2-1}$                       [ Let $e^{i\frac{4\pi}{11}}=t]$

                                                  $=\dfrac{-t(1-t)}{(t-1)(t+1)}$

                                                  $=\dfrac{-t}{t+1}$

                                                  $=\dfrac{-(\cos\frac{4\pi}{11}+i\sin\dfrac{4\pi}{11})}{\cos\dfrac{4\pi}{11}+i\sin\dfrac{4\pi}{11}+1}$

Let $a=\cos\dfrac{4\pi}{11}=a$ and $b=\sin\dfrac{4\pi}{11}$

                                                  $=\left(\dfrac{a+ib}{(a+1)+ib}\right)\times\dfrac{(a+1)-ib}{(a+1)-ib}$

                                                   $=-\dfrac{(1+ib)(a+1)-ib}{[(a+1)+ib][(a+1)-ib]}$

                                                   $=-\dfrac{(a(a+1)+b^2)+i(b(a+1)-ab)}{(a+1)^2+b^2}$

                                                   $=-\dfrac{a(a+1)+b^2}{(a+1)^2+b^2}$           [ Taking real part only ]

                                                   $=-\dfrac{a^2+b^2+a}{a^2+b^2+1+2a}$

                                                   $=-\dfrac{1+a}{2+2a}$

                                                   $=-\dfrac{(1+a)}{2(1+a)}$

                                                   $=\dfrac{-1}{2}$

If $x = \cos  \theta + i  \sin  \theta$ the value of $x^n + \dfrac{1}{x^n}$ is

  1. $2 \cos n \theta$

  2. $2 i \sin n \theta$

  3. $2 \sin n \theta$

  4. $2 i \cos n \theta$


Correct Option: A
Explanation:
$x=\cos \theta+i\sin \theta$
Applying Euler's form
$x=\cos \theta+i\sin \theta=e^{i\theta}$.
Hence 
$x^{n}=e^{in\theta}$. 
Similarly 
$\dfrac{1}{x}=\bar{x}=\cos \theta-i\sin \theta=e^{-i\theta}$
Hence 
$\dfrac{1}{x^{n}}=e^{-in\theta}$.
Hence 
$x^{n}+\dfrac{1}{x^{n}}=e^{in\theta}+e^{-in\theta}$
$=\cos n\theta+i\sin n\theta+\cos n\theta-i\sin n\theta$
$=2\cos n\theta$.

If $\alpha, \beta$ are the roots of the equation $u^2-2u+2=0$ and if $\cot\theta=x+1$, then $[(x+\alpha)^n-(x+\beta)^m]/[\alpha-\beta]$ is equal to

  1. $\displaystyle \frac {\sin n\theta}{\sin^n\theta}$

  2. $\displaystyle \frac {\cos n\theta}{\cos^n\theta}$

  3. $\displaystyle \frac {\sin n\theta}{\cos^n\theta}$

  4. $\displaystyle \frac {\cos n\theta}{\sin^n\theta}$


Correct Option: A
Explanation:

${ u }^{ 2 }-2u+2=0$
$\Longrightarrow \quad u=1\pm i$
So,$\alpha =1+i\quad and\quad \beta =1-i$
Now given that,$x=\cot { \theta  } -1$
so,$\displaystyle \frac { { (x+\alpha ) }^{ n }-{ (x+\beta ) }^{ n } }{ \alpha -\beta  } =\frac { { (\cot { \theta  } -1+1+i) }^{ n }-{ (\cot { \theta  } -1 }+1-i)^{ n } }{ 2i } \ \ $
$=\displaystyle \frac { { (\cot { \theta  } +i) }^{ n }-(\cot { \theta  } -i)^{ n } }{ 2i } =\frac { { (\cos { \theta  } +i\sin { \theta  } ) }^{ n }-{ (\cos { \theta  } -\sin { \theta  } ) }^{ n } }{ ({ \sin { \theta  }  })^{ n }(2i) } \ \ $
$=\displaystyle \frac { { e }^{ (in\theta ) }-{ e }^{ -(in\theta ) } }{ ({ \sin { \theta ) }  }^{ n }2i } \ \ $
=$\displaystyle \frac { (\cos { (n\theta ) } +i\sin { (n\theta )) } -(\cos { (n\theta ) } -i\sin { (n\theta )) }  }{ ({ \sin { \theta ) }  }^{ n }2i } =\frac { \sin { (n\theta ) }  }{ { (\sin { \theta ) }  }^{ n } } \ \ $

If $z _{1}$ and $\bar {z} _{1}$ represent adjacent of a regular polygon of $n$ sides with centre at the origin & if $\dfrac{Im\ z _{1}}{Re\ z _{1}}=\sqrt{2}-1$ then the value of $n$ is equal to:

  1. $8$

  2. $12$

  3. $16$

  4. $24$


Correct Option: A

What is the real part of $(\sin x + i \cos x)^{3}$ where $i = \sqrt {-1}$?

  1. $-\cos 3x$

  2. $-\sin 3x$

  3. $\sin 3x$

  4. $\cos 3x$


Correct Option: B
Explanation:
${ (\sin x+i\cos x) }^{ 3 }={ \sin }^{ 3 }x-i{ \cos }^{ 3 }x+3i\sin x\cos x(\sin x+i\cos x)$ 
$={ \sin }^{ 3 }x-i{ \cos }^{ 3 }x+3i{ \sin }^{ 2 }x\cos x-3\sin x\cos^{ 2 }x$
$={ \sin }^{ 3 }x-3\sin x\cos^{ 2 }x+i(3{ \sin }^{ 2 }x\cos x-{ \cos }^{ 3 }x)$
Real part is ${ \sin }^{ 3 }x-3\sin x\cos^{ 2 }x$
$=\sin x({ \sin }^{ 2 }x-3\cos^{ 2 }x)$
$=\sin x(-3+3{ \sin }^{ 2 }x+{ \sin }^{ 2 }x)$ 
$=\sin x(-3+4{ \sin }^{ 2 }x)$
$=-(3\sin x-4{ \sin }^{ 3 }x)$
$=-\sin3x$

If $(\cos  \theta  + i  \sin  \theta)(\cos  2 \theta 
+ i  \sin  2  \theta) ... (\cos  n  \theta + i  \sin  n  \theta) = 1$, then the value of $\theta$ is , $m\in N$  

  1. $4m\pi$

  2. $\displaystyle \frac{2m\pi}{n(n+1)}$

  3. $\displaystyle \frac{4m\pi}{n(n+1)}$

  4. $\displaystyle \frac{m\pi}{n(n+1)}$


Correct Option: C
Explanation:

Changing the above expression to Eular's form, we get
$e^{i\theta}e^{2i\theta}e^{3i\theta}...e^{in\theta})=1$
$e^{i(\theta+2\theta+3\theta+...n\theta}=1$
$e^{i\cfrac{n(n+1)}{2}\theta}=e^{2m\pi}$
Therefore, simplifying we get
$\dfrac{n(n+1)}{2}\theta=2m\pi$
$\theta=\dfrac{4m\pi}{n(n+1)}$

Statement 1: The product of all values of $(cos\alpha+i sin \alpha)^{\frac {3}{5}}$ is $cosn 3\alpha+i sin 3\alpha$.
Statement 2: The product of fifth roots of unity is 1.

  1. Both the statements are true, and Statement 2 is the correct explanation for Statement 1.

  2. Both the statements are true, but Statement 2 is not the correct explanation for Statement 1.

  3. Statement 1 is true and Statement 2 is false.

  4. Statement 1 is false and Statement 2 is true.


Correct Option: B
Explanation:

Let
$z=(cos\theta+isin\theta)^{3}$
Taking the fifth root, we get
$z^{\dfrac{1}{5}}=(cos\theta+isin\theta)^{\dfrac{3}{5}}=x$
Now there will be $5$, corresponding values of $x$.
Product if all the $5$ values will be
$x^{5}$
$=(cos\theta+isin\theta)^{3}$
$=cos3\theta+isin3\theta$ ... using De-Moivre's rule.
Now consider $x^{n}=1$
Hence if $n$ is odd, the nth roots of unity will be
$1,a _{1},\overline{a _{1}},a _{2},\overline{a _{2}}....$
Now $a _{1}.\overline{a _{1}}=|a _{1}|^{2}=1$
$a _{2}.\overline{a _{2}}=|a _{2}|^{2}=1$
:
:
Hence one root will be one, and the rest $n-1$ roots will occur in pair with its conjugate.
Hence product will be $1$.
Substituting, $n=5$, we get the roots as
$1,a _{1},\overline{a _{1}},a _{2},\overline{a _{2}}$
Hence product if all the roots will be $1$. And sum of all the roots will be $0$.
Both the statements are true, but Statement 2 is not the correct explanation for Statement 1.

If $z _1$ and $z _2$ are the complex roots of the equation $(x-3)^3+1 = 0$, then $z _1 + z _2$ equals to 

  1. 1

  2. 3

  3. 5

  4. 7


Correct Option: D
Explanation:

$cis\left( \theta  \right) =\cos { \theta  } +i\sin { \theta  } $
De Moivre's Theorem for fractional power:
${ \left( cis\theta  \right)  }^{ \frac { 1 }{ n }  }=cis\left( \frac { 2k\Pi +\theta  }{ n }  \right) $

${ \left( x-3 \right)  }^{ 3 }+1=0$
$\Longrightarrow x=3+{ \left( cis\left( \Pi  \right)  \right)  }^{ \frac { 1 }{ 3 }  }$
$x=3+{ \left( cis\left( \frac { 2k\Pi +\Pi  }{ 3 }  \right)  \right)  }$      ...{De Moivre's Theorem}
Where, $k=0,1,2$
for  $k=0$,
$x _{ 1 }=3+cis\left( \frac { \Pi  }{ 3 }  \right)$ 

for $k=1$,
$x _{ 2 }=3+cis\left( \Pi  \right) $

for $k=2,$
$x _{ 3 }=3+cis\left( \frac { 5\Pi  }{ 3 }  \right) $

$\Longrightarrow { x } _{ 1 }+{ x } _{ 3 }=6+cis\left( \frac { \Pi  }{ 3 }  \right) +cis\left( \frac { 5\Pi  }{ 3 }  \right) \ \Longrightarrow { x } _{ 1 }+{ x } _{ 3 }=7$
 
Ans: D

If $\left ( 2+z \right )^{6}+\left ( 2-z \right )^{6}=0$ and $\omega =\dfrac{2+z}{2-z}$

  1. $\displaystyle \omega =e^{i}\tfrac{\left (2p+1 \right )\pi }{6},p=0,1,2,3,4,5$

  2. $\displaystyle z=\frac{2\left ( \omega -1 \right )}{\omega +1}$

  3. $\displaystyle \omega = ( -1 )^(\frac{1}{6})$

  4. All of these


Correct Option: D
Explanation:

$\displaystyle { \left( 2+z \right)  }^{ 6 }{ +\left( 2-z \right)  }^{ 6 }=0\quad \quad &amp; \quad w=\frac { 2+z }{ 2-z } $

$\displaystyle \Rightarrow { \left( \frac { 2+z }{ 2-z }  \right)  }^{ 6 }=-1\ \Rightarrow { w }^{ 6 }=-1$

$\displaystyle { \therefore \quad w=\left( -1 \right)  }^{ \frac { 1 }{ 6 }  }$

$\displaystyle \because \quad \frac { 2+z }{ 2-z } =w\ \Rightarrow 2\left( w-1 \right) =z\left( w+1 \right) $

$\displaystyle \therefore \quad z=\frac { 2\left( w-1 \right)  }{ w+1 } $

$\displaystyle { \because \quad w=\left( -1 \right)  }^{ \frac { 1 }{ 6 }  }$

$\displaystyle w={ \left( \cos { \pi  } +i\sin { \pi  }  \right)  }^{ \frac { 1 }{ 6 }  }=\cos { \left( \frac { 2p\pi +\pi  }{ 6 }  \right) +i } \sin { \left( \frac { 2p\pi +\pi  }{ 6 }  \right)  } $       ..{De Moivre's Theorem}

Where$ p=0,1,2,3,4,5.$

$\displaystyle \Rightarrow w={ e }^{ i\frac { \left( 2p+1 \right) \pi  }{ 6 }  }$
Hence, option 'D' is correct.

Given $z$ is a complex number with modulus $1$. Then the equation $\dfrac{(1+ia)}{(1-ia)}$ = $z$ has

  1. all roots real and distinct

  2. two real and one imaginary

  3. three roots real and one imaginary

  4. one root real and three imaginary


Correct Option: A
Explanation:

$\displaystyle { \left( \frac { 1+ia }{ 1-ia }  \right)  }^{ 4 }=z\quad $         ...(1)
$\displaystyle &amp; \quad \left| z \right| =1$
$\displaystyle z=cisA=\cos { A } +i\sin { A } $
Substitute $z$  in equation (1)
$\displaystyle { \left( \frac { 1+ia }{ 1-ia }  \right)  }={ cisA }^{ \frac { 1 }{ 4 }  }=cis\frac { 2k\pi +A }{ 4 } $       ...{De Moivre's Theorem}
where $ k=0,1,2,3$

Let $\displaystyle B=\frac { 2k\pi +A }{ 4 } $

$\displaystyle \Longrightarrow ia=\frac { -1+cisB }{ 1+cisB } =\frac { \sin { \frac { B }{ 2 } \left( i\cos { \frac { B }{ 2 }  } -\sin { \frac { B }{ 2 }  }  \right)  }  }{ \cos { \frac { B }{ 2 } \left( \cos { \frac { B }{ 2 }  } +i\sin { \frac { B }{ 2 }  }  \right)  }  } $

$\displaystyle \Longrightarrow ia=\frac { i\sin { \frac { B }{ 2 } \left( \cos { \frac { B }{ 2 }  } +i\sin { \frac { B }{ 2 }  }  \right)  }  }{ \cos { \frac { B }{ 2 } \left( \cos { \frac { B }{ 2 }  } +i\sin { \frac { B }{ 2 }  }  \right)  }  } $

$\displaystyle \Longrightarrow a=\tan { \frac { B }{ 2 }  } $

Therefore roots are real and distinct.

Ans: A

If n is a natural number$ \ge$ 2, such that $z^n = (z+ 1)^n$, then 

  1. roots of equation lie on a straight line parallel to the y-axis

  2. roots of equation lie on a straight line parallel to the x-axis

  3. sum of the real parts of the roots is -[(n-1)/2]

  4. none of these


Correct Option: A,C
Explanation:

$\displaystyle { z }^{ n }={ \left( z+1 \right)  }^{ n }$   where, $n\ge 2$     ...(1)

$\displaystyle \Rightarrow { \left( \frac { z+1 }{ z }  \right)  }^{ n }=1=\cos { 0 } +i\sin { 0 } $

$\displaystyle \Rightarrow \frac { z+1 }{ z } ={ \left( \cos { 0 } +i\sin { 0 }  \right)  }^{ \frac { 1 }{ n }  }$

$\displaystyle \Rightarrow \frac { z+1 }{ z } =\cos { \frac { 2k\Pi  }{ n }  } +i\sin { \frac { 2k\Pi  }{ n }  } $      ...{De Moivre's Theorem}

$\displaystyle \Rightarrow z=\frac { -1 }{ 1-\cos { \frac { 2k\Pi  }{ n }  } -i\sin { \frac { 2k\Pi  }{ n }  }  } \quad =\frac { -1 }{ 2\sin { \frac { k\Pi  }{ n } \left( \sin { \frac { k\Pi  }{ n } -i } \cos { \frac { k\Pi  }{ n }  }  \right)  }  } =\frac { -1\left( \sin { \frac { k\Pi  }{ n } +i } \cos { \frac { k\Pi  }{ n }  }  \right)  }{ 2\sin { \frac { k\Pi  }{ n }  }  } $

$\displaystyle \Rightarrow z=\frac { -\left( 1+i\cot { \frac { k\Pi  }{ n }  }  \right)  }{ 2 } $

Where$ k=1,2,3,....,n-1 $     ..{Since at k=0, z is not defined}

$\because \quad Re\left( z \right) $ is constant.
Therfore roots of ${ z }^{ n }={ \left( z+1 \right)  }^{ n }$ lie on straight line parellel to y-axis.

$\displaystyle \because \quad z=\frac { -\left( 1+i\cot { \frac { k\Pi  }{ n }  }  \right)  }{ 2 } $ and $k=1,2,3,....,(n-1)$

Sum of $\displaystyle Re\left( z \right) $= $-\frac { \left( n-1 \right)  }{ 2 } $.

Ans: A,C

For positive integers $\displaystyle n _{1}$ and $\displaystyle n _{2}$ the value of the expression $\displaystyle (1+i)^{n _{1}}+(1+i^{3})^{n _{1}}+(1+i^{5})^{n _{2}}+(1+i^{2})^{n _{2}}$ where
$\displaystyle i= \sqrt{-1}$ is a real number iff

  1. $\displaystyle n _{1}= n _{2}$

  2. $\displaystyle n _{2}= n _{2}-1$

  3. $\displaystyle n _{1}= n _{2}+1$

  4. $\displaystyle \forall n _{1}$ and $\displaystyle n _{2}$


Correct Option: D
Explanation:

$(1+i)^{n _{1}}  = $ $  ^{n _{1}}C _{0} + ^{n _{1}}C _{1} i +^{n _{1}}C _{2} i^2 + ......+^{n _{1}}C _{n _{1}} i^{n _{1}}$ --------(1)


$(1+i^3)^{n _{1}}  =(1-i)^{n _{1}}=  $ $ ^{n _{1}}C _{0} - ^{n _{1}}C _{1} i +^{n _{1}}C _{2} i^2 - ......+^{n _{1}}C _{n _{1}} i^{n _{1}}$--------(2)

$(1+i^5)^{n _{2}}  =(1+i)^{n _{2}}=  $ $ ^{n _{2}}C _{0} + ^{n _{2}}C _{1} i +^{n _{2}}C _{2} i^2 + ......+^{n _{2}}C _{n _{2}} i^{n _{2}}$--------(3)

$(1+i^7)^{n _{2}}  =(1-i)^{n _{2}}=  $ $ ^{n _{2}}C _{0} - ^{n _{2}}C _{1} i +^{n _{2}}C _{2} i^2 - ......+^{n _{2}}C _{n _{2}} i^{n _{2}}$--------(4)

Adding (1),(2),(3) and (4),

$(1+i)^{n _{1}} +(1+i^3)^{n _{1}} +(1+i^5)^{n _{2}} +(1+i^7)^{n _{2}}$ 
$= 2(^{n _{1}}C _{0} +^{n _{1}}C _{2}i^2 +^{n _{1}} C _{4}i^4 +...........)+2(^{n _{2}}C _{0} +^{n _{2}}C _{2}i^2 +^{n _{2}} C _4 i^4+...........)$

$ = 2(^{n _{1}}C _{0} -^{n _{1}}C _{2} +^{n _{1}} C _{4} +...........)+2(^{n _{2}}C _{0} -^{n _{2}}C _{2} +^{n _{2}} C _4 +...........)$
$\Rightarrow$  As there are only even powers of $i$, the expression is real for all  positive integers $n _{1}$ and $n _{2}$

If ${ x }^{ 6 }={ \left( 4-3i \right)  }^{ 5 }$, then the product of all of its roots is (where $\displaystyle \theta =-\tan ^{ -1 }{ \frac { 3 }{ 4 }  } $)

  1. ${ 5 }^{ 5 }\left( \cos { 5\theta  } +i\sin { 5\theta  }  \right) $

  2. $-{ 5 }^{ 5 }\left( \cos { 5\theta  } +i\sin { 5\theta  }  \right) $

  3. ${ 5 }^{ 5 }\left( \cos { 5\theta  } -i\sin { 5\theta  }  \right) $

  4. $-{ 5 }^{ 5 }\left( \cos { 5\theta  } -i\sin { 5\theta  }  \right) $


Correct Option: B
Explanation:

$\displaystyle { x }^{ 6 }={ \left( 4-3i \right)  }^{ 5 }\Rightarrow { x }^{ 6 }={ 5 }^{ 6 }\left( \frac { 4 }{ 5 } -\frac { 3i }{ 5 }  \right) ={ 5 }^{ 5 }{ \left( \cos { \theta  } +i\sin { \theta  }  \right)  }^{ 5 }$

where $\displaystyle \theta =-\tan ^{ -1 }{ \frac { 3 }{ 4 }  } ={ 5 }^{ 5 }\left( \cos { 5\theta  } +i\sin { 5\theta  }  \right) $
$\displaystyle x={ 5 }^{ 5/6 }{ \left( \cos { 5\theta  } +i\sin { 5\theta  }  \right)  }^{ 1/6 }={ 5 }^{ 5/6 }\left[ \cos { \left( \frac { 2k\pi +5\theta  }{ 6 }  \right)  } +i\sin { \left( \frac { 2k\pi +5\theta  }{ 6 }  \right)  }  \right] $
${ x } _{ 1 }{ x } _{ 2 }{ x } _{ 3 }...{ x } _{ 6 }={ 5 }^{ 5 }\left( \cos { \left( 5\pi +5\theta  \right)  } +i\sin { \left( 5\pi +5\theta  \right)  }  \right) \ ={ 5 }^{ 5 }\left( -\cos { 5\theta  } -i\sin { 5\theta  }  \right) =-{ 5 }^{ 5 }\left( \cos { 5\theta  } +i\sin { 5\theta  }  \right) $

If $C _{o},C _{1},C _{2}...C _{n}$ are the Binomial coefficient in the expansion of $\left ( 1+x \right )^{n}$ then which is not correct

  1. $C _{0}-C _{2}+C _{4}-C _{6}+...=2\tfrac{n}{2}\cos \frac{n\pi }{4}$

  2. $C _{1}-C _{3}+C _{5}+...=2\tfrac{n}{2}\sin \frac{n\pi }{4}$

  3. $C _{1}+C _{5}+C _{9}+C _{13}+...=\tfrac{1}{2}\left ( 2^{n-1}+2\tfrac{n}{2}\sin \frac{n\pi }{4} \right )$

  4. None of these


Correct Option: D
Explanation:

By Binomial theorem
${ \left( 1+x \right)  }^{ n }={ C } _{ 0 }+{ C } _{ 1 }x+{ C } _{ 2 }{ x }^{ 2 }+{ C } _{ 3 }{ x }^{ \ 3 }+{ C } _{ 4 }{ x }^{ 4 }+...$      ...(1)

Substitute $x=i$ in equation (1), we get 
$\Rightarrow { \left( 1+i \right)  }^{ n }={ C } _{ 0 }+{ C } _{ 1 }i-{ C } _{ 2 }-{ C } _{ 3 }i+{ C } _{ 4 }+.....$       ...(2)

Substitute $x=-i$ in equation (1), we get
${ \left( 1-i \right)  }^{ n }={ C } _{ 0 }-{ C } _{ 1 }i-{ C } _{ 2 }+{ C } _{ 3 }i+{ C } _{ 4 }+.....$      ...(3)

Adding (2) & (3), we have
$\Rightarrow 2\left( { C } _{ 0 }{ -C } _{ 2 }{ +C } _{ 4 }+.... \right) ={ \left( 1+i \right)  }^{ n }+{ \left( 1-i \right)  }^{ n }$

$\Rightarrow { C } _{ 0 }{ -C } _{ 2 }{ +C } _{ 4 }+....=\dfrac { { \left( 1+i \right)  }^{ n }+{ \left( 1-i \right)  }^{ n } }{ 2 } ={ 2 }^{ \dfrac { n }{ 2 }  }\left( \dfrac { { \left( \cos { \dfrac { \pi  }{ 4 }  } +i\sin { \dfrac { \pi  }{ 4 }  }  \right)  }^{ n }+{ \left( \cos { \dfrac { \pi  }{ 4 }  } -i\sin { \dfrac { \pi  }{ 4 }  }  \right)  }^{ n } }{ 2 }  \right) $

$\Rightarrow { C } _{ 0 }{ -C } _{ 2 }{ +C } _{ 4 }+....={ 2 }^{ \dfrac { n }{ 2 }  }\left( \dfrac { \cos { \dfrac { n\pi  }{ 4 }  } +i\sin { \dfrac { n\pi  }{ 4 } +\cos { \dfrac { n\pi  }{ 4 }  } -i\sin { \dfrac { n\pi  }{ 4 }  }  }  }{ 2 }  \right) $

$\Rightarrow { C } _{ 0 }{ -C } _{ 2 }{ +C } _{ 4 }+....={ 2 }^{ \dfrac { n }{ 2 }  }\cos { \dfrac { n\pi  }{ 4 }  } $

Subtracting (2) & (3), we have
$\Rightarrow 2i\left( { C } _{ 1 }{ -C } _{ 3 }{ +C } _{ 5 }+.... \right) ={ \left( 1+i \right)  }^{ n }-{ \left( 1-i \right)  }^{ n }$

$\Rightarrow { C } _{ 1 }{ -C } _{ 3 }{ +C } _{ 5 }+....=\dfrac { { \left( 1+i \right)  }^{ n }-{ \left( 1-i \right)  }^{ n } }{ 2i } ={ 2 }^{ \dfrac { n }{ 2 }  }\left( \dfrac { { \left( \cos { \dfrac { \pi  }{ 4 }  } +i\sin { \dfrac { \pi  }{ 4 }  }  \right)  }^{ n }-{ \left( \cos { \dfrac { \pi  }{ 4 }  } -i\sin { \dfrac { \pi  }{ 4 }  }  \right)  }^{ n } }{ 2i }  \right) $

$\Rightarrow { C } _{ 1 }{ -C } _{ 3 }{ +C } _{ 5 }+....={ 2 }^{ \dfrac { n }{ 2 }  }\left( \dfrac { \cos { \dfrac { n\pi  }{ 4 }  } +i\sin { \dfrac { n\pi  }{ 4 } -\cos { \dfrac { n\pi  }{ 4 }  } +i\sin { \dfrac { n\pi  }{ 4 }  }  }  }{ 2i }  \right) $

$\Rightarrow { C } _{ 1 }{ -C } _{ 3 }{ +C } _{ 5 }+.....={ 2 }^{ \dfrac { n }{ 2 }  }\sin { \dfrac { n\pi  }{ 4 }  } $        ...(4)

Substitute $x=1$ in equation (1), we get
$\Rightarrow { C } _{ 0 }+{ C } _{ 1 }{ +C } _{ 2 }+{ C } _{ 3 }+{ C } _{ 4 }+.....\quad ={ 2 }^{ n }$          ...(5)

Substitute $x=-1$ in equation (1), we have
$\ \Rightarrow { C } _{ 0 }{ -C } _{ 1 }{ +C } _{ 2 }-{ C } _{ 3 }+{ C } _{ 4 }+.....\quad =0$       ...(6)

Subtracting (5) & (6), we get
$\Rightarrow 2\left( { C } _{ 1 }+{ C } _{ 3 }+{ C } _{ 5 }+..... \right) ={ 2 }^{ n }$
$\Rightarrow { C } _{ 1 }+{ C } _{ 3 }+{ C } _{ 5 }+.....\quad ={ 2 }^{ n-1 }$       ...(7)

Adding (4) & (7), we get
$\Rightarrow 2\left( { C } _{ 1 }+{ C } _{ 5 }+{ C } _{ 9 }+..... \right) ={ 2 }^{ n-1 }+{ 2 }^{ \dfrac { n }{ 2 }  }\sin { \dfrac { n\pi  }{ 4 }  } $

$\Rightarrow { C } _{ 1 }+{ C } _{ 5 }+{ C } _{ 9 }+.....=\dfrac { 1 }{ 2 } \left( { 2 }^{ n-1 }+{ 2 }^{ \dfrac { n }{ 2 }  }\sin { \dfrac { n\pi  }{ 4 }  }  \right) $
Hence, option 'D' is correct.

If $ x+\dfrac{1}{x}=2\cos \theta \   and \ y+\dfrac{1}{y}=2\cos \phi$  then which of the following is not correct?

  1. $\displaystyle \frac{x}{y} +\frac{y}{x}=2\cos \left ( \theta -\phi \right )$

  2. $x^{m}y^{n}=\cos \left ( m\theta +n\phi \right )+i\sin \left ( m\theta +n\phi \right )$

  3. $x^{m}y^{n}+x^{-m}y^{-n}=2\cos \left ( m\theta +n\phi \right )$

  4. None of these


Correct Option: C
Explanation:

$x+\dfrac { 1 }{ x } =2\cos { \theta  } \quad &amp; \quad y+\dfrac { 1 }{ y } =2\cos { \phi  } \ $

$\Rightarrow x=\cos { \theta  } +i\sin { \theta  } =cis\theta \ \quad &amp; \quad y=\cos { \phi  } +i\sin { \phi  } =cis\phi \ $

$\dfrac { x }{ y } +\dfrac { y }{ x } =\dfrac { cis\theta  }{ cis\phi  } +\dfrac { cis\phi  }{ cis\theta  } =cis\left( \theta -\phi  \right) +cis\left( -\theta +\phi  \right) $

$\therefore \quad \dfrac { x }{ y } +\dfrac { y }{ x } =2\cos { \left( \theta -\phi  \right)  } $

${ x }^{ m }{ y }^{ n }={ \left( cis\theta  \right)  }^{ m }{ \left( cis\phi  \right)  }^{ n }=\left( cism\theta  \right) \left( cisn\phi  \right) $          ...De Moivre's Theorem}

$\therefore \quad { x }^{ m }{ y }^{ n }=cis\left( m\theta +n\phi  \right) =\cos { \left( m\theta +n\phi  \right)  } +i\sin { \left( m\theta +n\phi  \right)  } $

${ x }^{ -m }{ y }^{ -n }={ \left( cis\theta  \right)  }^{ -m }{ \left( cis\phi  \right)  }^{ -n }=\left( cis\left( -m\theta  \right)  \right) \left( cis\left( -n\phi  \right)  \right) \ \therefore \quad { x }^{ -m }{ y }^{ -n }=cis\left( -m\theta -n\phi  \right) =\cos { \left( m\theta +n\phi  \right)  } -i\sin { \left( m\theta +n\phi  \right)  } \ $

$\therefore \quad { x }^{ m }{ y }^{ n }+{ x }^{ -m }{ y }^{ -n }=2\cos { \left( m\theta +n\phi  \right)  } $


Let $\mathrm{z}=\cos\theta+\mathrm{i}\sin\theta$. Then the value of $\displaystyle \sum _{\mathrm{m}=1}^{15}{\rm Im}(\mathrm{z}^{2\mathrm{m}-1})$ at $\theta =2^{\mathrm{o}}$ is

  1. $\displaystyle \frac{1}{\sin 2^{\mathrm{o}}}$

  2. $\displaystyle \frac{1}{3\sin 2^{\mathrm{o}}}$

  3. $\displaystyle \frac{1}{2\sin 2^{\mathrm{o}}}$

  4. $\displaystyle \frac{1}{4\sin 2^{\mathrm{o}}}$


Correct Option: D
Explanation:

$\mathrm{z}=\cos\theta+\mathrm{i}\sin\theta$
$\Rightarrow z^{2m-1} = \cos(2m-1)\theta+i\sin (2m-1)\theta$


Let $X = \displaystyle \sum _{\mathrm{m}=1}^{15}{\rm Im}(\mathrm{z}^{2\mathrm{m}-1})$

$\therefore \mathrm{X}=\sin\theta+\sin 3\theta+\ldots+\sin 29\theta$


$ 2(sin\theta)X=2\sin\theta\sin\theta + 2\sin\theta\sin3\theta...........+2\sin\theta\sin29\theta$
$\Rightarrow  2(\sin\theta)\mathrm{X}=1-\cos 2\theta+\cos 2\theta-\cos 4\theta+\ldots+\cos 28\theta-\cos 30\theta$

$\displaystyle \therefore  \mathrm{X}=\frac{1-\cos 30\theta}{2\sin\theta}=\frac{1}{4\sin 2^{\mathrm{o}}}$

- Hide questions